0 Daumen
832 Aufrufe

Aufgabe:

Zeigen Sie

\( \lim \limits_{n \rightarrow \infty} \sqrt[n]{n}=1 \)


Problem/Ansatz:

In der Lösung bzw in der Erklärung zum Beweis steht folgendes:

Quelle: "n-te Wurzel von n" auf https://de.wikibooks.org/wiki/Mathe_f%C3%BCr_Nicht-Freaks:_Grenzwert:_Beispiele

blob.png


Ich verstehe diesen Beweis nicht ganz. Warum muss man Zeigen, dass $$n$$ kleiner ist, als eine Teilsumme von $$(\epsilon + 1)^n$$ ? Und woher weiß ich überhaupt, dass das tatsächlich so ist, und das nicht eine größere Teilsumme benötigt wird? Würden größere Teilsummen auch gültig sein? Und wie sehe das aus, wenn man den Grenzwert aus der n-ten Wurzel von n + 1 oder n^2 zeigen wollen würde? (Welcher logischerweise auch 1 wäre)

Avatar von

1 Antwort

0 Daumen
 
Beste Antwort

Aloha :)

Eine Folge \((a_n)\) konvergiert gegen den Grenzwert \(a\), wenn es für alle \(\varepsilon\in\mathbb R^{>0}\) ein \(n_0\in\mathbb N\) gibt, sodass für alle \(n\ge n_0\) gilt: \(|a_n-a|<\varepsilon\).

In den Beweis wurde dies auf die Forderung \(n\stackrel!<(1+\varepsilon)^n\) zurückgeführt. In dem Folgenden geht es dann darum, ein \(n_0\) zu finden, ab dem diese Forderung für alle weiteren \(n\) gültig ist.

Ich finde den Beweis auch eher verwirrend und umständlich. Mit der Bernoulli-Ungleichung$$(1+x)^n\ge1+nx\quad\text{für }x\ge-1\;;\;n\in\mathbb N_0$$erhält man schnell folgende Abschätzung:

$$\left(1+\frac{1}{\sqrt n}\right)^n\ge1+\frac{n}{\sqrt n}=1+\sqrt n>\sqrt n=n^{1/2}\quad\implies$$$$\sqrt[n]{n}=n^{\frac{1}{n}}=\left(n^{1/2}\right)^{\frac{2}{n}}<\left(\left(1+\frac{1}{\sqrt n}\right)^n\right)^{\frac{2}{n}}=\left(1+\frac{1}{\sqrt n}\right)^2=1+\frac{2}{\sqrt n}+\frac 1n\le1+\frac{3}{\sqrt n}$$

Wählen wir nun ein \(\varepsilon>0\), so gilt:$$\left|\sqrt[n]{n}-1\right|\le\left|1+\frac3{\sqrt n}-1\right|=\frac3{\sqrt n}\stackrel!<\varepsilon\Longleftrightarrow\frac{9}{n}<\varepsilon^2\Longleftrightarrow n>\frac{9}{\varepsilon^2}$$Für alle \(n\ge n_0\) mit \(n_0=\left\lceil\frac{9}{\varepsilon^2}\right\rceil\) gilt also \(|\sqrt[n]{n}-1|<\varepsilon\). Damit ist der Grenzwert \(1\) bestätigt.

Avatar von 148 k 🚀

Ein anderes Problem?

Stell deine Frage

Willkommen bei der Mathelounge! Stell deine Frage einfach und kostenlos

x
Made by a lovely community